Verbal problems from the *free* official practice tests and
problems from mba.com
deep.within
Students
 
Posts: 3
Joined: Sat May 01, 2010 2:04 am
 

CR - television penetration

by deep.within Thu Jun 17, 2010 3:24 am

For the first time in history, more televisions than people can be found in American households. According to recent research, the average household has 2.55 residents and contains 2.73 televisions. However, by employing such costly manufacturing processes as plasma technology and flat screens, televisions are becoming too expensive for the typical consumer. As a result, the average number of residents per household will again surpass the number of televisions.

Which of the following, if true, most strongly supports the argument above?

A. House parties at which numerous individuals gather to view popular television shows on one television set have increased three hundred percent during the past year.
B. More than one million legal immigrants enter the United States each year.
C. New devices such as video-enabled personal digital assistants and music players are increasingly purchased for use as a primary source of information and entertainment.
D. As new technologies become more commonplace, manufacturing and retail costs normally decline.
E. As a result of technological advances, new televisions are increasingly enabled with some features, such as Internet browsing, traditionally associated with other household devices.



**SPOILER ALERT - ANSWER AHEAD**

1. My question is specifically for option B - if the no. of legal immigrants increase, won't that lead to an increase in the average number of residents per household as well?
2. The answer is C - Can someone help me understand how that is the right answer? Even if television ceases to be the primary source for info. etc., it will not lead to a disposal of the televisions already existing. So even if the number of televisions goes down in the future as this option suggests, the average no. of televisions per household is not going to decrease unless there is an increase in the no. of people -- can someone help me understand what's the flaw in my reasoning?

Thanks.
deep.within
Students
 
Posts: 3
Joined: Sat May 01, 2010 2:04 am
 

Re: CR - television penetration

by deep.within Sun Jun 20, 2010 7:02 am

Could one of the instructors help me out with this one please? It's from MGMAT's prep material.

Thanks!
mahesh.s009
Students
 
Posts: 11
Joined: Sat Jul 04, 2009 7:19 pm
 

Re: CR - television penetration

by mahesh.s009 Sun Jun 20, 2010 3:51 pm

My two cents :

Read the option B one more time. It happens each year and it is not a one time activity that is going to happen only this year. It would have happened last year as well so this can neither strengthen nor weaken the argument here.
parthak.dey
Students
 
Posts: 3
Joined: Thu Jun 17, 2010 8:56 pm
 

Re: CR - television penetration

by parthak.dey Mon Jun 21, 2010 1:34 am

What i understood:

A & B both are out of the scope statements. Although i believe corrct answer mentioned by you is "C". I would have picked "E" because as i believe the cost of the goods are increasing and a product with integrated feature will always be a wise purchse.

I will wait for the expert comment from Instructors.
Varshneya
Forum Guests
 
Posts: 5
Joined: Mon Dec 01, 2008 12:01 pm
 

Re: CR - television penetration

by Varshneya Thu Jun 24, 2010 5:13 am

my two cents worth...

1) Immigrants does not refer to residents. Resident is someone like a permanent resident or a citizen. I believe GMAT expects us to know this basic definition.

2) If other technology are primary source, then people will buy lesser TV. So, if the population remains same or increases, TV per household should decrease.

I have noticed a lot of CR mistakes are becuase we dont know the definition of these minor things like residents, consideration and so on.. CR is becoming more like reading comprehension.
Varshneya
Forum Guests
 
Posts: 5
Joined: Mon Dec 01, 2008 12:01 pm
 

Re: CR - television penetration

by Varshneya Thu Jun 24, 2010 5:19 am

I think it assumes in the passage that the number of household increases..That is why the conclusion is number of person per household will increase as a result.
RonPurewal
Students
 
Posts: 19744
Joined: Tue Aug 14, 2007 8:23 am
 

Re: CR - television penetration

by RonPurewal Mon Jul 19, 2010 2:41 am

when you do "strengthen" questions, like this one, sometimes you will encounter two answers that both seem to strengthen the argument.

if you have more than one choice that seems to strengthen/weaken an argument:
1) if one of them is MORE DIRECTLY RELATED to the argument, then pick that one.
otherwise,
2) if one of them makes MORE PROBABLE ASSUMPTIONS, then pick that one.


in any strengthen/weaken problem, at least one of these differences should be observable.

let's look at the two answer choices discussed by the original poster:

(b)
in order for this answer choice to strengthen the argument, you actually have to make at least one of the following two assumptions:
* the immigrants will have more residents per household than does the general population; or
* the immigrants will move into some already existing households.
you are assuming that the immigrants will actually increase the number of people PER HOUSEHOLD in the country (not the number of people in the country overall!!). if that is true, then one of the above assumptions must be true.
the second of these assumptions is extremely unlikely, and the first is basically a 50/50 shot -- we have no information in the passage about the density with which the new immigrants live in their households, so we would just be guessing here.

also, you would have to assume that the new immigrants will not purchase more televisions per household that is characteristic for existing households. like the above, this is another 50/50 guess; we have no idea how many televisions will be purchased by these immigrants, nor do we have any sort of evidence upon which to base a guess.

therefore, choice (b) is extremely dubious, because it is based upon not just one, but TWO assumptions that are basically random guesses -- whose opposites could be true just as easily as they could!

takeaway:
do not postulate some random hypothetical -- which could be false just as easily as it could be true -- and then build an argument upon that random hypothetical.

if your random hypothetical is EXTREMELY likely to be true -- or, equivalently, if the opposite of your random hypothetical is extremely unlikely -- then it is more ok to build an argument upon such a hypothetical.


2. The answer is C - Can someone help me understand how that is the right answer? Even if television ceases to be the primary source for info. etc., it will not lead to a disposal of the televisions already existing. So even if the number of televisions goes down in the future as this option suggests, the average no. of televisions per household is not going to decrease unless there is an increase in the no. of people -- can someone help me understand what's the flaw in my reasoning?


here you also have to make an assumption -- namely, that people will dispose of television sets at a non-negligible rate.

this assumption is much better justified than are the two assumptions in the choice above. in the choice above, each of the two necessary assumptions could be false just as easily as it could be true.
in this choice, on the other hand, it's extremely unlikely that the necessary assumption could be false -- i.e., it's extremely unlikely that everybody is just going to hold on to their old televisions indefinitely. while this is possible, it's certainly much less likely than that people will gradually dispose of the old televisions.
rkkarthik
Course Students
 
Posts: 15
Joined: Tue Dec 22, 2009 5:20 am
 

Re: CR - television penetration

by rkkarthik Thu Nov 18, 2010 7:11 pm

Ron,

"Right on the Money" - Excellent explanation! Thanks

Karthik
RonPurewal
Students
 
Posts: 19744
Joined: Tue Aug 14, 2007 8:23 am
 

Re: CR - television penetration

by RonPurewal Thu Nov 25, 2010 6:21 am

rkkarthik Wrote:Ron,

"Right on the Money" - Excellent explanation! Thanks

Karthik


sure.

this problem is an excellent illustration of the annoying distinction between
* problems on which you select the answer that MUST be true, such as "find the assumption" and "draw the conclusion" questions;
and
* problems on which you select the MOST PROBABLE answer, such as strengthen/weaken or "explain the discrepancy" problems.

notice that, in the latter of these, you usually have to employ real-world common sense to figure out the most probable assumptions, or most probable outcomes, of certain statements -- you are NOT talking about things that can be proved from the statements at hand.

this is one illustration of why it's really important to be aware of the particular problem type on which you're working.

every problem type is different!
s.ashwin.rao
Students
 
Posts: 40
Joined: Sat Nov 27, 2010 4:38 pm
 

Re: CR - television penetration

by s.ashwin.rao Sun Mar 06, 2011 9:15 am

Ron,
Could you kindly also exlain why A is wrong? Is it because A talks of House Parties and hence out of scope?
RonPurewal
Students
 
Posts: 19744
Joined: Tue Aug 14, 2007 8:23 am
 

Re: CR - television penetration

by RonPurewal Tue Mar 08, 2011 6:29 am

s.ashwin.rao Wrote:Ron,
Could you kindly also exlain why A is wrong? Is it because A talks of House Parties and hence out of scope?


whoa, no.
on strengthening/weakening problems, the correct answer will ALWAYS be outside the scope of the passage in at least one way!
think about it -- if you need to strengthen or weaken an argument in real life, you need to bring in more information from outside to either support or contravene the argument. if you stay within the existing scope of the argument, you can't strengthen or weaken it, since you're just treating the material that has already been discussed.

rather, (a) is incorrect because it just doesn't strengthen the argument.
the main idea of the argument is that people are going to buy fewer televisions; you need something that strongly supports the claim that people are going to buy fewer televisions.
choice (c) definitely supports this claim; if people are purchasing devices in order to replace televisions, then it's clear that television sales will decrease.
choice (a) doesn't give us any reason to think that overall television purchases will decrease -- it's inconclusive. in fact, one could make a strong case that television purchases will actually increase if there is such a massive increase in parties dedicated to watching television!
sauravkohli4
Students
 
Posts: 3
Joined: Wed Dec 31, 1969 8:00 pm
 

Re: CR - television penetration

by sauravkohli4 Tue Dec 06, 2011 8:18 am

Ron-I still dont understand why (C) is correct. The arguement states that people will stop buying tvs because tvs are getting costlier.(C) does not bring in the cost factor.

Also, you mentioned in your last post -"if people are purchasing devices in order to replace televisions, then it's clear that television sales will decrease." But (C) does not suggest that people are buying these devices to replace television. It just says that they use them as a primary source of entertainment. So we can say that people are buying this to replace primary source of entertainment/information(which can be reading books,palying basketball...).

I was confused b/w A & C(picked A ) but i dont like both of them.I think i am lost here.Pls help
RonPurewal
Students
 
Posts: 19744
Joined: Tue Aug 14, 2007 8:23 am
 

Re: CR - television penetration

by RonPurewal Sat Dec 10, 2011 5:56 am

sauravkohli4 Wrote:Ron-I still dont understand why (C) is correct. The arguement states that people will stop buying tvs because tvs are getting costlier.(C) does not bring in the cost factor.


you are missing the point of strengthen/weaken problems -- the whole point is that you don't have to follow the same logical path in order to strengthen or weaken a statement.
for instance:
if i move to new york, i will have to walk around more and drive less. therefore, i will lose weight if i move to new york.
--> this argument would be weakened by the statement "in new york, it will be much harder for me to buy and eat healthy foods than it is where i live now".
--> this argument would be strengthened by the statement "in new york, it will be much easier for me to buy and eat healthy, low-calorie foods than it is where i live now".

neither of the above ideas deals with the amount of walking that i have to do, but their bearing on the FINAL CONCLUSION OF THE ARGUMENT (the idea that i'm going to lose weight) should be clear.
[/quote]
chembeti_aravind
Forum Guests
 
Posts: 8
Joined: Fri Dec 23, 2011 2:14 pm
 

Re: CR - television penetration

by chembeti_aravind Wed Jan 18, 2012 12:58 pm

RonPurewal Wrote:
sauravkohli4 Wrote:Ron-I still dont understand why (C) is correct. The arguement states that people will stop buying tvs because tvs are getting costlier.(C) does not bring in the cost factor.


you are missing the point of strengthen/weaken problems -- the whole point is that you don't have to follow the same logical path in order to strengthen or weaken a statement.
for instance:
if i move to new york, i will have to walk around more and drive less. therefore, i will lose weight if i move to new york.
--> this argument would be weakened by the statement "in new york, it will be much harder for me to buy and eat healthy foods than it is where i live now".
--> this argument would be strengthened by the statement "in new york, it will be much easier for me to buy and eat healthy, low-calorie foods than it is where i live now".

neither of the above ideas deals with the amount of walking that i have to do, but their bearing on the FINAL CONCLUSION OF THE ARGUMENT (the idea that i'm going to lose weight) should be clear.
[/quote]

I would say like this.
The conclusion we need to accept from the argument is televisions sales will be less.
Now, we know that TVs are primarily used for entertainment. If TVs sales come down, people must buy some other devices for the same purpose. And this is what is explained in C.

But note that C is the right answer only in the given set of options. Instead of mentioning PDAs, C could also say something else in similar lines, for e.g., popularity for Theater Art is going to increase, etc.
RonPurewal
Students
 
Posts: 19744
Joined: Tue Aug 14, 2007 8:23 am
 

Re: CR - television penetration

by RonPurewal Mon Jan 23, 2012 3:17 am

chembeti_aravind Wrote:But note that C is the right answer only in the given set of options. Instead of mentioning PDAs, C could also say something else in similar lines, for e.g., popularity for Theater Art is going to increase, etc.


this is true -- and, in fact, it's true for all strengthening/weakening questions. there is essentially never going to be only one way to strengthen or weaken an argument.
so, your job is not "find THE strengthener/weakener"; it's "find WHICH OF THE FOLLOWING CHOICES is a strengther/weakener".